Question

Stock Z has a beta of 0.5 and an expected return of 8%. If Treasury Bills...

Stock Z has a beta of 0.5 and an expected return of 8%. If Treasury Bills currently return 1% and the expected return on the S&P 500 is 7%, is this stock correctly priced, underpriced, or overpriced? Graph the security market line and Stock Z. Label all relevant details. Explain the concept of market efficiency using your graph.

Homework Answers

Answer #1

Kindly do let me know in case you have any queries.

Know the answer?
Your Answer:

Post as a guest

Your Name:

What's your source?

Earn Coins

Coins can be redeemed for fabulous gifts.

Not the answer you're looking for?
Ask your own homework help question
Similar Questions
#24 Stock A has a beta of 1.2 and an expected return of 12%. Stock B...
#24 Stock A has a beta of 1.2 and an expected return of 12%. Stock B has a beta of 0.7 and an expected return of 8%. If the risk-free rate is 2% and the market risk premium is 8%, what is true about the two stocks? A. Stock A is underpriced and stock B is overpriced B. Both stocks are underpriced C. Stock A is overpriced and stock B is underpriced D. Both stocks are correctly priced E. Both...
You believe that Alpha stock which has a beta of 1.32 will return 16.0% this coming...
You believe that Alpha stock which has a beta of 1.32 will return 16.0% this coming year. The market is expected to return 11.4% and T-bills return 3.8%. According to CAPM, which one of these statements is correct given this information? Multiple Choice The stock is currently underpriced The stock plots to the left of the market on a security market line graph The stock plots below the security market line The stock is currently underpriced. The stock plots to...
Stock A has an expected return of 13% and a standard deviation of 22%, while Stock...
Stock A has an expected return of 13% and a standard deviation of 22%, while Stock B has an expected return of 15% and a standard deviation of 25%. If an investor is less risk-averse, they will be likely to choose… A. Stock A B. Stock B Stock A has a beta of 1.8 and an expected return of 12%. Stock B has a beta of 0.7 and an expected return of 7%. If the risk-free rate is 2% and...
Q#25 The beta of Ricci Co.'s stock is 2.8, whereas the risk-free rate of return is...
Q#25 The beta of Ricci Co.'s stock is 2.8, whereas the risk-free rate of return is 8 percent. If the expected return on the market is 17 percent, then what is the expected return on Ricci Co.? Suppose this stock has an expected return of 40%. Is this security properly priced? A) 42.80%, overpriced B) 33.20%, overpriced C) 42.80%, underpriced D) 33.20%, underpriced
Stock A has an expected return of 18.6 percent and a beta of 1.2. Stock B...
Stock A has an expected return of 18.6 percent and a beta of 1.2. Stock B has an expected return of 15 percent and a beta of 0.9. Both stocks are correctly priced and lie on the Security market Line (SML). What is the reward-to-risk ratio for stock A? (6marks) (Use the simplest way to calculate)
Stock Y has a beta of 1.5 and an expected return of 14.2 percent. Stock Z...
Stock Y has a beta of 1.5 and an expected return of 14.2 percent. Stock Z has a beta of 0.85 and an expected return of 10.7 percent.     Required: If the risk-free rate is 4.6 percent and the market risk premium is 7.1 percent, are these stocks correctly priced?        Stock Y undervalued or overvalued?     Stock Z undervalued or overvalued?  
Security X has an expected rate of return of 13% AND A BETA OF 1.15. The...
Security X has an expected rate of return of 13% AND A BETA OF 1.15. The risk-free is 5%, and the market expected rate of return is 15%. According to the capital asset pricing model, security X is _______. a. fairly priced b. overpriced c. underpriced d none of these answers (I need assistance on how to calculate and conclude.)
Stock A has a beta of 1.25 and an expected return of 14%. Stock B has...
Stock A has a beta of 1.25 and an expected return of 14%. Stock B has a beta of 0.75 and an expected return of 10%. Now, if you construct a portfolio of stock A and Treasury Bills that has the same risk as the market index, what will be your portfolio’s expected return? Assume all stocks are fairly priced.
Stock X has a beta of 1.55 and is expected to generate the following returns given...
Stock X has a beta of 1.55 and is expected to generate the following returns given the three different economic states, boom, normal and recession. State Probability Expected return Boom 50% 45% Good 30% 25% Poor 20% -20% The market risk premium is 12% and the risk-free rate is 7%.   Based on the above information, the stock is (underpriced, overpriced, correctly priced) , therefore we should (buy, sell, hold)  the stock. This stock plots (on, above, below)  SML.
A particular asset has a beta of 1.2 and an expected return of 10%. The expected...
A particular asset has a beta of 1.2 and an expected return of 10%. The expected return on the market portfolio is 13% and the risk-free rate is 5%. The share is: -- Hint: Compare Expected Return to Required Return A. overpriced B. underpriced C. appropriately priced D. There is not enough information to answer the question. Previous